www.vorhilfe.de
- Förderverein -
Der Förderverein.

Gemeinnütziger Verein zur Finanzierung des Projekts Vorhilfe.de.
Hallo Gast!einloggen | registrieren ]
Startseite · Mitglieder · Impressum
Forenbaum
^ Forenbaum
Status VH e.V.
  Status Vereinsforum

Gezeigt werden alle Foren bis zur Tiefe 2

Navigation
 Startseite...
 Suchen
 Impressum
Das Projekt
Server und Internetanbindung werden durch Spenden finanziert.
Organisiert wird das Projekt von unserem Koordinatorenteam.
Hunderte Mitglieder helfen ehrenamtlich in unseren moderierten Foren.
Anbieter der Seite ist der gemeinnützige Verein "Vorhilfe.de e.V.".
Partnerseiten
Weitere Fächer:

Open Source FunktionenplotterFunkyPlot: Kostenloser und quelloffener Funktionenplotter für Linux und andere Betriebssysteme
Forum "Uni-Stochastik" - Momente
Momente < Stochastik < Hochschule < Mathe < Vorhilfe
Ansicht: [ geschachtelt ] | ^ Forum "Uni-Stochastik"  | ^^ Alle Foren  | ^ Forenbaum  | Materialien

Momente: Frage (beantwortet)
Status: (Frage) beantwortet Status 
Datum: 13:43 Mo 21.05.2012
Autor: Fry

Eingabefehler: "{" und "}" müssen immer paarweise auftreten, es wurde aber ein Teil ohne Entsprechung gefunden (siehe rote Markierung)
Eingabefehler: "{" und "}" müssen immer paarweise auftreten, es wurde aber ein Teil ohne Entsprechung gefunden (siehe rote Markierung)


Hallo zusammen,
könnte mir jemand erklären,
warum aus $E[e^{tX}]<\infty$ für ein $t>0$ folgt, dass $E|X|^n<\infty$ für alle $n\in\N$ (zumindest hab ich das so in einem Skript gefunden...)

Kann mir höchstens erklären, dass dies für $n=1$ gilt.
Nach der Jensen Ungleichung gilt $e^{tE|X|}\le E(e^{t|X|)$
(wobei hier $X$ leider mit Betragsstrichen steht)
Dann ist $E|X|\le \frac{1}{t}\log E(e^{t|X|)$.

Hätte jemand einen Tipp für mich?


LG
Fry



        
Bezug
Momente: Antwort
Status: (Antwort) fertig Status 
Datum: 14:59 Mo 21.05.2012
Autor: Gonozal_IX

Hiho,

die Aussage ist offensichtlich ohne weitere Angaben falsch.

Bspw. für $X [mm] \equiv -\infty$ [/mm] gilt für jedes t>0 [mm] $E[e^{tX}] [/mm] = E[0] = 0 < [mm] \infty$, [/mm] aber [mm] $E[|X|^n] [/mm] = [mm] \infty$ [/mm] für [mm] $n\in\IN$ [/mm]

MFG,
Gono.

Bezug
Ansicht: [ geschachtelt ] | ^ Forum "Uni-Stochastik"  | ^^ Alle Foren  | ^ Forenbaum  | Materialien


^ Seitenanfang ^
ev.vorhilfe.de
[ Startseite | Mitglieder | Impressum ]